The Praxis® Study Companion

Middle School: Social Studies 5089

www.ets.org/praxis Welcome to the Praxis® Study Companion

Welcome to the Praxis® Study Companion

Prepare to Show What You Know You have been working to acquire the knowledge and skills you need for your teaching career. Now you are ready to demonstrate your abilities by taking a Praxis® test.

Using the Praxis® Study Companion is a smart way to prepare for the test so you can do your best on test day. This guide can help keep you on track and make the most efficient use of your study time.

The Study Companion contains practical information and helpful tools, including:

• An overview of the Praxis tests • Specific information on thePraxis test you are taking • A template study plan • Study topics • Practice questions and explanations of correct answers • Test-taking tips and strategies • Frequently asked questions • Links to more detailed information

So where should you start? Begin by reviewing this guide in its entirety and note those sections that you need to revisit. Then you can create your own personalized study plan and schedule based on your individual needs and how much time you have before test day.

Keep in mind that study habits are individual. There are many different ways to successfully prepare for your test. Some people study better on their own, while others prefer a group dynamic. You may have more energy early in the day, but another test taker may concentrate better in the evening. So use this guide to develop the approach that works best for you.

Your teaching career begins with preparation. Good luck!

Know What to Expect

Which tests should I take? Each state or agency that uses the Praxis tests sets its own requirements for which test or tests you must take for the teaching area you wish to pursue.

Before you register for a test, confirm your state or agency’s testing requirements atwww.ets.org/praxis/states .

How are the Praxis tests given? Praxis tests are given on computer. Other formats are available for test takers approved for accommodations (see page 45).

The Praxis® Study Companion 2 Welcome to the Praxis® Study Companion

What should I expect when taking the test on computer? When taking the test on computer, you can expect to be asked to provide proper identification at the test center. Once admitted, you will be given the opportunity to learn how the computer interface works (how to answer questions, how to skip questions, how to go back to questions you skipped, etc.) before the testing time begins. Watch the What to Expect on Test Day video to see what the experience is like.

Where and when are the Praxis tests offered? You can select the test center that is most convenient for you. The Praxis tests are administered through an international network of test centers, which includes Prometric® Testing Centers, some universities, and other locations throughout the world.

Testing schedules may differ, so see thePraxis web site for more detailed test registration information at www. ets.org/praxis/register.

The Praxis® Study Companion 3 Table of Contents

Table of Contents

The Praxis® Study Companion guides you through the steps to success

1. Learn About Your Test...... 5 Learn about the specific test you will be taking

2. Familiarize Yourself with Test Questions...... 10 Become comfortable with the types of questions you’ll find on thePraxis tests

3. Practice with Sample Test Questions...... 14 Answer practice questions and find explanations for correct answers

4. Determine Your Strategy for Success...... 28 Set clear goals and deadlines so your test preparation is focused and efficient

5. Develop Your Study Plan...... 31 Develop a personalized study plan and schedule

6. Review Study Topics...... 35 Review study topics with questions for discussion

7. Review Smart Tips for Success...... 43 Follow test-taking tips developed by experts

8. Check on Testing Accommodations...... 45 See if you qualify for accommodations to take the Praxis test

9. Do Your Best on Test Day...... 46 Get ready for test day so you will be calm and confident

10. Understand Your Scores...... 48 Understand how tests are scored and how to interpret your test scores

Appendix: Other Questions You May Have ...... 50

The Praxis® Study Companion 4 Step 1: Learn About Your Test

1. Learn About Your Test Learn about the specific test you will be taking

Middle School: Social Studies (5089)

Test at a Glance

Test Name Middle School: Social Studies Test Code 5089 Time 2 hours Number of Questions 90 selected-response questions (Part A); 3 constructed-response questions short-answer essays) (Part B) Format Selected-response and constructed-response questions Weighting Selected-response: 75% of total score; constructed response: 25% of total score Test Delivery Computer delivered Approximate Approximate Content Categories Number of Percentage of Questions Examination

I. History 22 19% VI I II. World History 18 15% II III. Government/Civics 17 14% V IV. Geography 17 14% III IV V. Economics 16 13% VI. Short Content Essays 3 25% 1. United States History, related to Government/Civics 2. World History, related to Geography 3. U.S. History, related to Economics or Geography OR World History, related to Economics or Government/Civics Pacing and In allocating time on this assessment, it is expected that about 90 minutes will be Special Tips spent on the selected-response section and about 30 minutes will be spent on the constructed-response section; the sections are not independently timed.

The Praxis® Study Companion 5 Step 1: Learn About Your Test

About This Test Test Specifications

The Middle School: Social Studies test assesses the Test specifications in this chapter describe the knowledge and skills necessary for a beginning knowledge and skills measured by the test. Study middle school social studies . The test is based topics to help you prepare to answer test questions on the N C S S National Standards for Social Studies can be found on page 35. and measures examinees’ knowledge and skills related to the following concepts: United States I. United States History History, World History, Government/Civics, Geography, A. Chronological Developments in United and Economics. Some of the selected-response and States History all of the short essay questions are interdisciplinary, reflecting the complex relationship among the 1. Understands basic North American geography social studies fields. Some questions are based and the peoples and cultures of North America prior to European colonization on interpreting stimulus material such as written passages, maps, charts, graphs, tables, cartoons, 2. Understands how and why European colonies diagrams, and photographs. in North America were founded and developed The three equally weighted short-answer/essay 3. Understands how European, African, and questions will focus on important historical events American Indian peoples interacted in North and issues, as well as on fundamental social studies America during the colonial period concepts. These questions, which should take about 4. Understands the origins of the American 10 minutes each to complete and will together Revolution and the challenges faced by the comprise 25 percent of the examinee’s score, will early republic emphasize the exercise of critical thinking skills, 5. Understands the context that led to the requiring the reading and interpreting of social studies writing and adoption of the United States materials (such as maps, charts, quotations); drawing Constitution and Bill of Rights and their impact inferences from such materials; and placing these on the political development of the early materials in their historical, geographical, political, and United States economic contexts. The 90 equally weighted selected- 6. Understands causes (e.g., Manifest Destiny, response questions will constitute 75 percent of the gold rush) and effects (e.g., displacement of examinee’s score. native peoples, economic expansion) of territorial expansion of the United States in the This test may contain some questions that will not 19th century count toward your score. 7. Understands causes and effects of 19th- century sectionalism, the Civil War, and Reconstruction 8. Understands the relationship among industrialization, urbanization, and immigration in the late 19th and early 20th centuries 9. Knows major political, economic, social, and cultural developments from the Progressive Era through the New Deal 10. Understands causes of United States participation in the First and Second World Wars and effects of the wars at home and abroad 11. Understands the origins, development, and effects of the Cold War both abroad and domestically

The Praxis® Study Companion 6 Step 1: Learn About Your Test

12. Understands social and political changes in the 5. Knows major causes and effects of growing United States from the 1940s to the 1970s (e.g., global interactions from 1200 to 1750 C.E. (e.g., Civil Rights movement, counterculture, Columbian Exchange, colonization, the Plague) conservatism) 6. Knows major causes and effects of revolutions, 13. Understands the impact of economic and nationalism, and imperialism from 1750 to technological changes in the latter half of the 1914 20th century (e.g., globalization, information 7. Knows major ideological, economic, and age) political causes and effects of the First and B. Major Themes in United States History Second World Wars and the Cold War (e.g., 1. Understands ongoing impacts of racial, Russian Revolution, decolonization) gender, and ethnic divisions throughout 8. Knows major developments of the post–Cold American history War world (e.g., growth of the globalized 2. Understands how participants in the political economy, rise of fundamentalism and process (e.g., presidents, the judiciary, nationalism) Congress, and political parties) engaged in B. Major Themes in World History politics and shaped policy 1. Understands how scientific inquiry, 3. Understands the emergence of the United technological innovations, and adaptations States as a world power and the evolving role shaped world societies of the United States in the world 2. Understands the role of major world religions 4. Understands major economic transformations in shaping societies and effecting major (e.g., in agriculture, business, and labor) that historical turning points (e.g., the spread of occurred in the United States Buddhism, the Crusades, Hinduism) 5. Understands causes and effects of changing 3. Knows the role of trade and other forms of patterns of immigration to the United States economic exchange (e.g., tribute, gift giving) and internal migration within the United States both within societies and in contacts between 6. Understands the struggle and achievements societies by individuals and groups for greater political 4. Knows major political ideologies that have and civil rights throughout United States influenced the organization of societies in the history modern world (e.g., fascism, liberalism, nationalism) II. World History 5. Understands major economic transformations that have affected world societies (e.g., A. Chronological Developments in World feudalism, spread of the market economy, History industrialization) 1. Knows the formation, organization, and 6. Understands the roles of both conflict (e.g., contributions of early civilizations in Africa, imperialism) and cooperation (e.g., United Europe, and Asia from 8000 B.C.E. to 1000 B.C.E. Nations) in shaping and transforming societies and in the Americas from 2000 B.C.E. to 1500 C.E. 7. Knows major demographic trends (e.g., urbanization, migration, population growth) in 2. Knows the formation, organization, and world history and their effects contributions of the classical civilizations of Greece, Rome, Persia, India, and China from 8. Knows how exploration (e.g., continental, 1000 B.C.E. to 500 C.E. overseas, outer space) affects human societies 3. Knows how civilizations changed as a result of 9. Understands how struggles for human rights invasions, trade, and the spread of Islam and and individual expression shaped various Christianity from 300 to 1400 C.E. societies 4. Knows causes and effects of the Renaissance, Reformation, and Enlightenment from 1400 to 1800 C.E.

The Praxis® Study Companion 7 Step 1: Learn About Your Test

III. Government/Civics IV. Geography A. United States Government and Civics A. Geographic Literacy 1. Understands the key concepts and ideas on 1. Understands relative and absolute location which the United States government is based and the physical and human characteristics of (e.g., popular sovereignty, separation of “place” powers, rule of law) 2. Understands how to utilize characteristics to 2. Understands federalism and the basic identify regions relationship between the states and the B. Map Skills national government 1. Understands and interprets various types of 3. Understands the origins, development, maps interpretations, and ongoing impact of the United States Constitution 2. Understands latitude and longitude 4. Understands the origin, role, and interactions 3. Understands spacial patterns and their of the three branches of the federal meanings government 4. Is familiar with recent technological sources of 5. Understands the development and political geographic information impact of civil rights and civil liberties in the C. Physical Geography United States 1. Demonstrates knowledge of various types of 6. Understands how the election process physical features on the Earth’s surface operates in the United States 2. Understands the interrelationship between the 7. Understands the role of political parties, environment and human activity interest groups and voluntary associations, and 3. Knows the physical processes related to the the media in the political process formation of geographic features 8. Understands the rights, responsibilities, and 4. Knows the basic patterns of climate, duties of citizens ecosystems, and natural resources B. Comparative Government and D. Human Geography International Relations 1. Knows spatial patterns of culture and 1. Understands the major characteristics of economic activities different political systems (e.g., democracy, republic, totalitarianism) 2. Understands basic population patterns 2. Understands the relationship between political 3. Understands basic patterns of land use systems and economic conditions 4. Understands patterns and interrelationships of migration, trade, and diffusion of ideas 5. Knows basic concepts of political geography 6. Understands how culture and cultural change, human adaptation, and diversity influence human behavior

The Praxis® Study Companion 8 Step 1: Learn About Your Test

V. Economics VI. Short Content Essays A. Microeconomics I The three equally weighted short-answer/essay 1. Knows the concepts of scarcity, choice, and questions are designed to test knowledge of history opportunity costs and social studies and the ability to define and clarify 2. Knows the differences among market, issues, judge information, and draw conclusions. command, and mixed economies In most cases, questions ask examinees to utilize B. Microeconomics II these abilities in demonstrating an understanding of 1. Demonstrates knowledge of types of market stimulus materials such as maps, graphs, charts, tables, structure and the characteristics and behavior cartoons, diagrams, quotations, or excerpts from of firms in perfect competition, monopoly, documents. oligopoly, and monopolistic competition Each of the three questions in the test will have both a 2. Demonstrates understanding of factor markets history and a social studies component. and the determination of income distributions and the returns to factors of production (wage, The three questions will be divided as follows: interest, rent, profits) C. Macroeconomics I 1. United States History, related to Government/Civics 1. Demonstrates understanding of gross 2. World History, related to Geography domestic product (G D P) and its components 3. United States History, related to Economics or 2. Demonstrates understanding of how Government/Civics unemployment is measured and its causes and 4. World History, related to Economics or consequences Government/Civics 3. Understands the principle of supply and OR demand 1. United States History, related to Government/Civics 4. Knows how government intervention and public policies affect the economy (e.g., 2. World History, related to Geography taxation, regulation) 3. United States History, related to Economics OR 5. Knows how consumer choices affect the Geography economy OR 6. Understands basic principles of personal finance (e.g., budgeting, interest, investment) 1. United States History, related to Government/Civics 7. Understands how the factors of production 2. World History, related to Geography (i.e., land, labor, capital, and entrepreneurship) 3. World History, related to Economics OR affect economic activity Government/Civics 8. Knows different measures of economic performance (e.g., G D P, unemployment, inflation) 9. Understands basic functions of money and how it circulates through the economy 10. Knows the interdependence of the global economy

The Praxis® Study Companion 9 Step 2: Familiarize Yourself with Test Questions

2. Familiarize Yourself with Test Questions Become comfortable with the types of questions you’ll find on thePraxis tests

The Praxis assessments include a variety of question types: constructed response (for which you write a response of your own); selected response, for which you select one or more answers from a list of choices or make another kind of selection (e.g., by clicking on a sentence in a text or by clicking on part of a graphic); and numeric entry, for which you enter a numeric value in an answer field. You may be familiar with these question formats from taking other standardized tests. If not, familiarize yourself with them so you don’t spend time during the test figuring out how to answer them.

Understanding Computer-Delivered Questions

Questions on computer-delivered tests are interactive in the sense that you answer by selecting an option or entering text on the screen. If you see a format you are not familiar with, read the directions carefully. The directions always give clear instructions on how you are expected to respond.

For most questions, you respond by clicking an oval to select a single answer from a list of answer choices.

However, interactive question types may also ask you to respond by: • Clicking more than one oval to select answers from a list of choices. • Typing in an entry box. When the answer is a number, you may be asked to enter a numerical answer. Some questions may have more than one place to enter a response. • Clicking check boxes. You may be asked to click check boxes instead of an oval when more than one choice within a set of answers can be selected. • Clicking parts of a graphic. In some questions, you will select your answers by clicking on a location (or locations) on a graphic such as a map or chart, as opposed to choosing your answer from a list. • Clicking on sentences. In questions with reading passages, you may be asked to choose your answers by clicking on a sentence (or sentences) within the reading passage. • Dragging and dropping answer choices into targets on the screen. You may be asked to select answers from a list of choices and drag your answers to the appropriate location in a table, paragraph of text or graphic. • Selecting answer choices from a drop-down menu. You may be asked to choose answers by selecting choices from a drop-down menu (e.g., to complete a sentence).

Remember that with every question you will get clear instructions.

Perhaps the best way to understand computer-delivered questions is to view the Computer-delivered Testing Demonstration on the Praxis web site to learn how a computer-delivered test works and see examples of some types of questions you may encounter.

The Praxis® Study Companion 10 Step 2: Familiarize Yourself with Test Questions

Understanding Selected-Response Questions Many selected-response questions begin with the phrase “which of the following.” Take a look at this example:

Which of the following is a flavor made from beans? (A) Strawberry (B) Cherry (C) Vanilla (D) Mint

How would you answer this question? All of the answer choices are flavors. Your job is to decide which of the flavors is the one made from beans.

Try following these steps to select the correct answer.

1) Limit your answer to the choices given. You may know that chocolate and coffee are also flavors made from beans, but they are not listed. Rather than thinking of other possible answers, focus only on the choices given (“which of the following”).

2) Eliminate incorrect answers. You may know that strawberry and cherry flavors are made from fruit and that mint flavor is made from a plant. That leaves vanilla as the only possible answer.

3) Verify your answer. You can substitute “vanilla” for the phrase “which of the following” and turn the question into this statement: “Vanilla is a flavor made from beans.” This will help you be sure that your answer is correct. If you’re still uncertain, try substituting the other choices to see if they make sense. You may want to use this technique as you answer selected-response questions on the practice tests.

Try a more challenging example The vanilla bean question is pretty straightforward, but you’ll find that more challenging questions have a similar structure. For example:

Entries in outlines are generally arranged according to which of the following relationships of ideas? (A) Literal and inferential (B) Concrete and abstract (C) Linear and recursive (D) Main and subordinate

You’ll notice that this example also contains the phrase “which of the following.” This phrase helps you determine that your answer will be a “relationship of ideas” from the choices provided. You are supposed to find the choice that describes how entries, or ideas, in outlines are related.

Sometimes it helps to put the question in your own words. Here, you could paraphrase the question in this way: “How are outlines usually organized?” Since the ideas in outlines usually appear as main ideas and subordinate ideas, the answer is (D).

The Praxis® Study Companion 11 Step 2: Familiarize Yourself with Test Questions

QUICK TIP: Don’t be intimidated by words you may not understand. It might be easy to be thrown by words like “recursive” or “inferential.” Read carefully to understand the question and look for an answer that fits. An outline is something you are probably familiar with and expect to teach to your students. So slow down, and use what you know.

Watch out for selected-response questions containing “NOT,” “LEAST,” and “EXCEPT” This type of question asks you to select the choice that does not fit. You must be very careful because it is easy to forget that you are selecting the negative. This question type is used in situations in which there are several good solutions or ways to approach something, but also a clearly wrong way.

How to approach questions about graphs, tables, or reading passages When answering questions about graphs, tables, or reading passages, provide only the information that the questions ask for. In the case of a map or graph, you might want to read the questions first, and then look at the map or graph. In the case of a long reading passage, you might want to go ahead and read the passage first, noting places you think are important, and then answer the questions. Again, the important thing is to be sure you answer the questions as they refer to the material presented. So read the questions carefully.

How to approach unfamiliar formats New question formats are developed from time to time to find new ways of assessing knowledge. Tests may include audio and video components, such as a movie clip or animation, instead of a map or reading passage. Other tests may allow you to zoom in on details in a graphic or picture.

Tests may also include interactive questions. These questions take advantage of technology to assess knowledge and skills in ways that standard selected-response questions cannot. If you see a format you are not familiar with, read the directions carefully. The directions always give clear instructions on how you are expected to respond.

QUICK TIP: Don’t make the questions more difficult than they are. Don’t read for hidden meanings or tricks. There are no trick questions on Praxis tests. They are intended to be serious, straightforward tests of your knowledge.

Understanding Constructed-Response Questions Constructed-response questions require you to demonstrate your knowledge in a subject area by creating your own response to particular topics. Essays and short-answer questions are types of constructed-response questions.

For example, an essay question might present you with a topic and ask you to discuss the extent to which you agree or disagree with the opinion stated. You must support your position with specific reasons and examples from your own experience, observations, or reading.

Take a look at a few sample essay topics:

• “Celebrities have a tremendous influence on the young, and for that reason, they have a responsibility to act as role models.” • “We are constantly bombarded by advertisements—on television and radio, in newspapers and magazines, on highway signs, and the sides of buses. They have become too pervasive. It’s time to put limits on advertising.” • “Advances in computer technology have made the classroom unnecessary, since students and teachers are able to communicate with one another from computer terminals at home or at work.”

The Praxis® Study Companion 12 Step 2: Familiarize Yourself with Test Questions

Keep these things in mind when you respond to a constructed-response question 1) Answer the question accurately. Analyze what each part of the question is asking you to do. If the question asks you to describe or discuss, you should provide more than just a list.

2) Answer the question completely. If a question asks you to do three distinct things in your response, you should cover all three things for the best score. Otherwise, no matter how well you write, you will not be awarded full credit.

3) Answer the question that is asked. Do not change the question or challenge the basis of the question. You will receive no credit or a low score if you answer another question or if you state, for example, that there is no possible answer.

4) Give a thorough and detailed response. You must demonstrate that you have a thorough understanding of the subject matter. However, your response should be straightforward and not filled with unnecessary information.

5) Reread your response. Check that you have written what you thought you wrote. Be sure not to leave sentences unfinished or omit clarifying information.

QUICK TIP: You may find that it helps to take notes on scratch paper so that you don’t miss any details. Then you’ll be sure to have all the information you need to answer the question.

For tests that have constructed-response questions, more detailed information can be found on page 23.

The Praxis® Study Companion 13 Step 3: Practice with Sample Test Questions

3. Practice with Sample Test Questions

Answer practice questions and find explanations for correct answers

Computer Delivery This test is available via computer delivery. To illustrate what the computer-delivered test looks like, the following sample question shows an actual screen used in a computer-delivered test. For the purposes of this guide, sample questions are provided as they would appear in a paper-delivered test.

The Praxis® Study Companion 14 Step 3: Practice with Sample Test Questions

Sample Test Questions

The sample questions that follow illustrate the kinds of questions on the test. They are not, however, representative of the entire scope of the test in either content or difficulty. Answers with explanations follow the questions. Directions: Each of the questions or incomplete statements below is followed by four suggested answers or completions. Select the one that is best in each case.

1. “Rather than having to pay for costly fortifications, armies, and navies, this country enjoyed free protection provided by nature. Furthermore, for more than a century, British subjects manned and paid for the costly navy that policed and defended the Atlantic and provided this country cost-free added 4. All of the following conclusions about woman security.” suffrage can be made on the basis of the map The country that benefited from the added above EXCEPT: security referred to above was which of the (A) Between 1890 and 1920, many western following? states had granted women equal (A) Germany suffrage. (B) Holland (B) Westward settlement inhibited the growth of equal suffrage up to 1920. (C) Sweden (C) In many states on the eastern coast, (D) The United States women were denied suffrage until the passage of the Nineteenth Amendment. 2. The majority of immigrants who arrived in the (D) In some of the midwestern states, United States before the Civil War came from women had voting rights. (A) Northern and Western Europe (B) Southern and Eastern Europe 5. For the greater part of the time humankind has existed on earth, people have obtained (C) Latin America and the Caribbean their food by which of the following means? (D) Asia and the Pacific Islands I. Hunting II. Gathering 3. “They made us many promises, more than I can remember, but they kept only one: they III. Agriculture promised to take our land, and they took it.” (A) I only The statement above was most probably (B) III only made by (C) I and II only (A) an African American (D) I and III only (B) a Greek American (C) a Polish American (D) a Native American

The Praxis® Study Companion 15 Step 3: Practice with Sample Test Questions

6. In the 19th century, European powers carved up large regions with little concern for ethnic or cultural bonds, thus splitting some groups and forcing others to live alongside groups with whom they had little in common. All of the following countries experienced ethnic strife after independence EXCEPT (A) Nigeria (B) Egypt 10. On the basis of the information given in the (C) India map above, which of the cities has the greatest potential for economic development? (D) Sudan (A) A

7. According to the United States Constitution, (B) B the President has the power to (C) C (A) negotiate treaties (D) D (B) amend the Constitution (C) impeach members of Congress 11. Workers in the United States widget industry (a small and specialized labor force) received (D) raise and support an army a substantial increase in wages, making the hourly cost of their wages and benefits the 8. Which of the following is most essential to the highest for blue-collar workers in the U.S. and successful functioning of a democracy? much higher than those of widget-producing workers in competing countries. (A) Three branches of government (B) Participating citizens The wage increase described above is most likely to result in an increase in the (C) Direct election of the head of government (A) profits of widget companies in the United States (D) Federalism (B) sales of products that are made with widgets in the United States 9. All of the following river valleys are densely populated EXCEPT the (C) number of widgets imported into the United States (A) Yangtze (D) number of widgets sold in the United (B) Amazon States (C) Nile (D) Indus

The Praxis® Study Companion 16 Step 3: Practice with Sample Test Questions

12. “I was adamant about getting fathers into the 15. The guest-worker program in western Europe labor room and into the delivery room. I was attracts workers primarily from insistent about fathers attending parenting (A) the United States classes. The only way I would take parents was as couples. I wrote an article for a family (B) Latin America magazine and encouraged them to put a (C) North Africa and Southwest Asia father holding a baby on the cover. Today we see fathers pushing baby strollers, carrying (D) Australia and New Zealand babies on slings. We see men doing commercials for diapers and showing tender 16. A similarity between the cities of Varanasi loving care. There have been tremendous (Banaras) in India and Mecca in Saudi Arabia changes. It is no longer considered is that both are ‘unmasculine’ to be affectionate.” (A) capitals of countries formerly colonized by the English The quote above is from a 1987 interview with an obstetrician who has been practicing (B) destinations for vast numbers of religious medicine for decades. The changes pilgrims mentioned by the obstetrician refer to (C) financial centers for a large fraction of changes in social the world’s economy (A) regulations (D) examples of modern urban planning (B) norms (C) policies 17. The Jakota Triangle is a region consisting of large cities, numerous exports, increasing (D) instincts consumption of raw materials, and global financial connections. Which of the following 13. The gross domestic product of a country is lists the principle countries of this region? defined as the total (A) France, Portugal, Spain (A) output of goods and services sold in the (B) Japan, South Korea, Taiwan country during a year (C) Cuba, Haiti, Jamaica (B) output of goods and services consumed in the country during a year (D) Iceland, Ireland, the United Kingdom (C) market value of all goods and services sold in the country during a year 18. Which of the following is an example of the constitutional system of checks and (D) market value of all final goods and balances? services produced in the country during a year (A) A Supreme Court ruling that upholds a state law 14. Which of the following will increase the (B) An interest group that lobbies Congress demand for teachers? to reject proposed legislation (A) Increasing the salaries of teachers (C) A state that lobbies Congress for increased federal funding (B) Raising the retirement age for teachers (D) The impeachment and removal of a (C) Lowering the student-teacher ratio federal official (D) Consolidating school districts

The Praxis® Study Companion 17 Step 3: Practice with Sample Test Questions

19. Which of the following is involved in the “Fellow citizens, pardon me, allow me to ask, appointment and confirmation process of why am I called upon to speak here today? federal judges in the United States? What have I, or those I represent, to do with your national independence? Are the great (A) The president principles of political freedom and of natural (B) The chief justice of the Supreme Court justice, embodied in that Declaration of (C) The state governors Independence, extended to us?... I am not included within the pale of this glorious (D) The sitting judges on the United States anniversary!... This Fourth of July is yours, not Courts of Appeals mine.”

20. In addition to the power of control over the 22. In the speech excerpted above, the African budget, Congress can control or influence the American leader Frederick Douglass bureaucracy by expresses his sentiments about the (A) bringing public opinion to bear on an (A) triumph of freedom over slavery in the agency Civil War (B) holding investigations into the activities (B) gap between democratic ideals and the of an agency reality of slavery (C) ordering the president to reform an (C) horrors and injustices he personally agency experienced under slavery (D) making an agency report directly to (D) relationship between republican ideals Congress and Reconstruction actions

21. A similarity between Brown v. Board of 23. Which of the following was a charge made by Education of Topeka (1954) and Miranda v. Senator Joseph McCarthy about dangers Arizona (1966) was that the rulings in both facing the United States in the 1950s? cases (A) There were a significant number of (A) affirmed and protected civil rights under communists in positions of authority in the Constitution the United States government. (B) involved the rights of those accused of (B) Most United States voters were crimes sympathetic to the communists. (C) involved the issue of Jim Crow laws (C) The United States economy was (D) tended to limit the power of the federal threatened by cheap foreign imports. government (D) An increase in the rate of illegal immigration was threatening the stability of United States society.

24. The greatest expansion of direct European colonial rule in Africa occurred in which of the following periods? (A) 1450–1500 (B) 1750–1800 (C) 1850–1900 (D) 1900–1950

The Praxis® Study Companion 18 Step 3: Practice with Sample Test Questions

25. Which of the following was an outcome of the 26. Which of the following best summarizes the Paris Peace Conference in 1919 at the contributions of Muslim scholars to the conclusion of the First World War, giving rise development of science in the period circa to tensions that would help lead to the 700–1400 C.E.? outbreak of the Second World War? (A) They worked in isolation from the (A) The Treaty of Versailles forced Germany Chinese and Indian scholars of the same to give up land, population, and period. important resources. (B) They made advances mostly in medicine (B) The Treaty of Versailles divided Germany and the life sciences. into eastern and western states. (C) Most of their works became known to (C) The British gave control of Palestine to European scholars only in the 19th and the Arabs living there. 20th centuries. (D) It was agreed to leave the Austrian (D) They preserved the scientific tradition of Empire intact. the ancient world and expanded it greatly by their own discoveries.

The Praxis® Study Companion 19 Step 3: Practice with Sample Test Questions

Answers to Sample Questions

1. The correct answer is (D). Of the countries 6. The correct answer is (B). Nigeria became named, only the United States enjoyed geographic independent in 1960, and in 1966 violent clashes isolation (“free protection provided by nature”). In between the Hausa people and the Igbo people addition, the British colonies that later became the led to the deaths of thousands of Igbo and the United States constituted the only listed area that establishment of the independent Igbo state of had been protected by the British navy. Biafra, later reabsorbed into Nigeria. India became independent in 1947. Ethnic strife between Hindus 2. The correct answer is (A). Prior to the Civil War, and Muslims preceded independence and led to immigration to the United States came almost the establishment of the Muslim state of Pakistan. exclusively from Northern and Western Europe, Strife between Hindus and Muslims continues in with large numbers of people coming from India, however. Sudan became independent in Ireland, Great Britain, Germany, and Scandinavia. 1956. Ethnic strife between the predominantly Arab The immigration wave from Southern and Eastern and Muslim North and the predominantly African Europe followed the Civil War, peaking in the late and Christian/animist South preceded 19th and early 20th centuries. Immigration from independence and led to two periods of civil war, Latin America and the Caribbean did not become from 1955 to 1972, and from 1983 to 2005. Egypt, significant numerically until the second half of the which became independent in 1922, is relatively 20th century. Immigration from Asia and the homogeneous in terms of ethnicity, and has not Pacific Islands was significant in the last quarter of experienced civil wars along ethnic lines. the 19th century and in the second half of the 20th century. 7. The correct answer is (A). The power to make treaties is specifically granted to the President in 3. The correct answer is (D). Of the ethnic groups Article II of the Constitution. Under Article V, the named, only the Native Americans were offered power to amend the Constitution is reserved to the promises (in the form of treaties) that were broken states and to Congress. Congress, under Article I, with the result of being deprived of their land. has the power to impeach the President, but the African Americans were deprived of their freedom reverse is not true. Also under Article I, Congress has and their rights, but were not generally offered the power to raise and support an army. promises or deprived of land. Greek and Polish Americans experienced discrimination, but were 8. The correct answer is (B)—without the offered no official promises and were not deprived participation of its citizens, a nation cannot have a of their land. successful democracy. While the United States has three branches of government, this number of 4. The correct answer is (B). As the map indicates, branches is not necessary for a successful most western states were early in granting women democracy. Many successful democracies, suffrage, while those of the east coast were slower including Great Britain, do not have direct election to do so, and those of the Midwest were split in of the head of government. Federalism is a their response. Since the states of the west were component of many, but not all, successful among the first to give women the right to vote, democracies. France, for example, does not have a westward settlement clearly did not inhibit the federal form of government. growth of equal suffrage. 9. The correct answer is (B). The valleys of the 5. The correct answer is (C), I and II only. The oldest Yangtze in China, the Nile in Egypt, and the Indus in known remains of homo sapiens have been dated India and Pakistan all support huge populations of at 75,000 to 115,000 years old. The earliest people, densely clustered along these evidence of agriculture dates from 10,000 years watercourses. The Amazon, on the other hand, ago. Until then, humans survived only by hunting flows for much of its length through very sparsely animals and gathering plants for food. populated rain forest. 10. The correct answer is (B). City B is at the confluence of two rivers and the crossroads of three major highways; since this is true of none of the other cities, City B has the greatest potential for economic development.

The Praxis® Study Companion 20 Step 3: Practice with Sample Test Questions

11. The correct answer is (C). The increase in the 17. The correct answer is (B). The European wages of widget workers has led to an increase in countries, (A) and (D), are not primarily exporting the cost of widgets produced in the United States. countries and only Ireland and the United This will almost inevitably lead to an increase in the Kingdom offer extensive financial connections. (C) price of domestically produced widgets which will, is not correct because Cuba, Haiti, and Jamaica as a in turn, lead to consumers seeking cheaper region do not have large cities, numerous exports, widgets—those produced in other countries. or an increasing consumption of raw materials. 12. The correct answer is (B). The obstetrician is 18. The correct answer is (D), since the discussing changes in people’s learned behavior impeachment process is a tool to curb the power made through education, encouragement, and of the executive branch. A Supreme Court ruling example. No actions by government or another upholding a state law is merely an affirmation of official body are mentioned, therefore eliminating the state’s powers. A state lobbying for increased regulations (A) and policies (C). Instincts (D) are not Congressional funding or an interest group learned behavior but rather are innate and would lobbying for a particular stand on legislation is not be affected by the obstetrician’s actions. merely affirming the power of the legislative body “Norms,” society’s often unwritten and unspoken and is not a check on its powers. rules, serve to guide and control proper and 19. The correct answer is (A). The president acceptable behavior and can be affected by the nominates federal judges, who are then confirmed type of actions that the obstetrician describes by the Senate. None of the other officials listed taking. above are involved in the process. 13. The correct answer is (D). This question is 20. The correct answer is (B). Government definitional; only (D) has the complete definition. agencies, as part of the executive branch, report to 14. The correct answer is (C). Lowering the the president, not Congress, and Congress does student-teacher ratio requires the creation of many not have the authority to order the president to sections of a class. Since additional teachers need reform an agency. However, Congress does have to be hired to accomplish the task, the demand for the power of regulatory oversight and the power teachers will increase. (A) is incorrect because to appropriate funds for agencies. Therefore, under increasing the salaries of teachers increases the the system of checks and balances, Congress can number of people willing to teach, but it may hold investigations into the activities of an agency. decrease the number of teachers that will be 21. The correct answer is (A). The Supreme Court demanded. (B) is incorrect; raising the retirement in Brown v. Board of Education of Topeka ruled that age for teachers limits the availability of vacancies. segregated schooling denied “equal protection of (D) is incorrect, since consolidating school districts the laws” guaranteed by the Fourteenth may decrease the demand for teachers. Amendment, while Miranda v. Arizona was based 15. The correct answer is (C). Guest workers are on the Fifth Amendment’s protection against solicited from nearby nations, often those with a self-incrimination. Only Miranda dealt with the colonial connection to the receiving nation. Since rights of the accused, and only Brown concerned guest workers serve as temporary workers, Jim Crow laws. Neither tended to limit federal proximity to the receiving country is important. power. Thus, although all have colonial ties to countries of 22. The correct answer is (B). It indicates that western Europe, the United States (A), Latin Douglass was using a rhetorical device to America (B), and Australia and New Zealand (D) emphasize the gap between the principles of would not be likely source countries for western freedom and justice championed in the Europe because of the great distance. Declaration of Independence and the continuing 16. The correct answer is (B). Varanasi (Banaras) is a existence of a slave system in the United States. holy city of the Hindus and the object of constant The quotation does not concern the Civil War, pilgrimages; Mecca is a holy city and principal Douglass’ personal experience, or Reconstruction. pilgrimage destination of Islam.

The Praxis® Study Companion 21 Step 3: Practice with Sample Test Questions

23. The correct answer is (A). Senator Joseph 25. The correct answer is (A). Germans believed McCarthy gained prominence based on charges that they had been cheated by the peace that communists had attained positions of settlement agreed to in the Treaty of Versailles, and influence in the United States government. He did this sense of resentment later fueled the rise of the not, however, claim that most United States voters Nazi Party. The Treaty of Versailles did not break supported communists. Nor did he focus on the Germany up into western and eastern parts (B); issues of imports or illegal immigration. that division took place after the end of the Second World War. The British did not give control 24. The correct answer is (C). The largest of Palestine to the Arabs living there (C). Rather, in expansion of African territory directly under the Balfour Declaration, Britain indicated that it European colonial control took place in the late “view[ed] with favour the establishment in 19th century, especially in the wake of the Berlin Palestine of a national home for the Jewish people.” Conference of 1884. During this period, Britain The Austrian empire was broken up into various substantially expanded its colonial territories in parts as a result of the peace conference, with West Africa (Nigeria), East Africa (Kenya, Uganda, Czechoslovakia, Yugoslavia, and Hungary created etc.), and southern Africa (Rhodesia, Botswana, in its place. etc.); France acquired large territories in West Africa (Mauritania, Senegal, Mali, Niger, etc.), Central 26. The correct answer is (D). Muslim scholars of Africa (Chad, Congo-Brazzaville, etc.), and North that period were largely responsible for the Africa (Tunisia); Germany created a substantial preservation of classical Greek and Roman texts colonial presence (Namibia, Tanganyika, Togo, etc.); and their eventual transmission to western Europe Portugal consolidated and expanded its African in the late Middle Ages. Under the generous colonies (Angola, Mozambique, etc.); and the patronage of the Abbasid caliphs and other Belgian king Leopold II acquired a huge personal Muslim rulers, Muslim scientists built on the fiefdom in central Africa (The Congo Free State). foundation of classical science with numerous original discoveries in fields such as mathematics, astronomy, physics, chemistry, and medicine.

The Praxis® Study Companion 22 Step 3: Practice with Sample Test Questions

Sample Test Questions Score of 2 • Shows an adequate understanding of the stimulus This section presents sample questions and responses (where appropriate) along with the standards used in scoring the responses. When you read these sample responses, keep in mind that • Provides a mostly accurate and complete response they will be less polished than if they had been developed at • Provides most of the analysis required by the home, edited, and carefully presented. Examinees do not question know what questions will be asked and must decide, on the Applies mostly appropriate subject matter knowledge spot, how to respond. Readers take these circumstances • into account when scoring the responses. • May contain significant errors

Readers will assign scores based on the following scoring Score of 1 guide. • Shows little understanding of the stimulus (where Scoring Guide appropriate) • Provides a basically inaccurate and incomplete Score of 3 response • Shows a thorough understanding of the stimulus • Provides little of the analysis required by the question (where appropriate) • Applies mostly inappropriate subject matter • Provides an accurate and complete response knowledge • Provides the analysis required by the question Score of 0 • Applies appropriate subject matter knowledge • A totally or almost completely incorrect response; or • May contain minor errors simply rephrases the question

The Praxis® Study Companion 23 Step 3: Practice with Sample Test Questions

Sample Question 1 Sample Response that Received a Score of 1

Brown v. Board of Education of Topeka The Supreme Court doctrine issued in 1954 was the 14th Amendment. The rationale given by the jus- “We come then to the question presented: Does tices was that separate did not mean equal. Minority segregation of children in public schools solely on students were not being educated by knowledgeable the basis of race, even though the physical facilities and well trained teachers. The facilities in which the and other ‘tangible’ factors may be equal, deprive the minority students were to learn in were not adequate. children of the minority group of equal educational The materials such as textbooks were not widely opportunities? We believe that it does.” available to minorities. The segregation was anything but equal. (A) What legal doctrine or principle, established in Plessy v. Ferguson (1896), did the Supreme Court reverse when it issued the 1954 ruling quoted above? (B) What was the rationale given by the justices for their 1954 ruling?

Sample Response that Received a Score of 3 In 1954, the Supreme Court ruled that separate facili- ties are inherently unequal. This reversed the principle of legal segregation that was established by Plessy v. Ferguson in 1896. In Plessy, the court ruled that sepa- rate facilities for whites and blacks were constitutional as long as the facilities were equal.

The rationale for the 1954 ruling was based on the belief that minorities who are separated from the majority culture through racial segregation will not have access to the same experiences, opportunities and privileges as the majority population. The jus- tices concluded that separate facilities could never be equal facilities and that legal segregation in public education would have to end.

The Praxis® Study Companion 24 Step 3: Practice with Sample Test Questions

Sample Question 2

The map above shows the routes of some early to trade with foreigners because they thought expeditions and explorations. Choose one of these the foreigners were inferior, the trade expeditions or explorations and then do the following. restrictions could not prevent the world from discovering China. Not long after, a Portuguese (A) Describe the route of the expedition or ship landed at China and in 1557 the exploration (its origin, destination, and any Portuguese made a settlement on the other information related to the route taken), southeastern coast of China at Macao, near and explain how geographic factors (terrain, Guanghou. Jesuit missionaries built missions ocean currents, prevailing winds) influenced there and began to convert Chinese to the course of the expedition or exploration. Christianity. As they were well-educated in (B) State the reason for the expedition or astronomy, math and arts, the Chinese learned exploration, and explain why the expedition or western astronomy and math. Europeans who exploration was important. wanted China’s teas, silk and porcelain brought sweet potatoes and corn from the Americas. Sample Response That Received a Score of 3 The expedition was important because it made China known to the world and paved the way (A) In the Ming Dynasty, the early emperors wanted for trade between China and foreign countries to display to the world the splendors of China. and an exchange of culture. Zheng He, a court ambassador, was ordered by his emperor to make voyages from China to Sample Response That Received a Score of 1 south-eastern Asia, India, Arabia and Africa. From 1405–1433, Zheng He made 7 voyages to Marco Polo began his exploration from Italy, came the Middle East and the eastern coast of Africa. down through the Mediterranean across Asia by land route. Marco was sent by his government in search Chinese merchant ships had been sailing in of spices and gold and the mapping the world. At this the Indian Ocean long before Zheng He’s time this was “the world” in the minds of the people. expeditions, and his voyages benefitted greatly Vasco da Gama took off where Marco Polo left off from the knowledge accumulated during these and succeeded in traveling through the West Indies earlier contacts. Zheng He planned the route of and around the Cape of Good Hope. Marco Polo, his voyages so that his fleet would benefit from like most other explorers, were commissioned by the seasonal southwestern and northeastern their governments to seek new lands for Imperialistic Monsoonal winds and the Monsoon-related purposes, seek slaves, seek spices which were high in oceanic currents (drifts) of the Indian Ocean. demand and seek prestige for that country for having (B) The reason for the expedition was to collect simply discovered something. Marco Polo succeeded gifts and to display to the world the wealth and in bringing “the Orient” to Italy. power of China (in the Ming Dynasty). Although the Ming emperors forbade Chinese merchants

The Praxis® Study Companion 25 Step 3: Practice with Sample Test Questions

Sample Question 3

The Praxis® Study Companion 26 Step 3: Practice with Sample Test Questions

(A) What do the two maps on page 26 reveal Sample Response That Received a Score of 1 about the changes in cotton production in the (A) From the 2 maps shown, cotton production in United States between 1811 and 1859? the United States more than quadrupled from (B) Based on your knowledge of United States 1811 to 1859. In 1811, cotton production in the history, briefly describe two key USA was limited to and developments—political, economic, or and some in , but in 1859, all of the technological—that brought about these South has been “overcome” by cotton changes. production. It has even stretched west to eastern part of Texas. By 1859, cotton Sample Response That Received a Score of 3 production which was formerly limited to 3 states has now spread to , (A) The two maps illustrate that cotton production northern Florida, , , was a minor production crop in 1811 and , and Eastern Texas. confined to mostly the mid-Atlantic states. It was harvested in the Carolinas with pockets in (B) In the 1811 period, cotton production was , Virginia and Georgia. Obviously, limited mostly due to limited labor. But as soon things had changed by 1859 with the most as slaves were brought from Africa, labor was southern states producing the bulk of the abundant and cotton production spread like cotton. wild fire. (B) One reason for the increase in cotton production between 1811–59 was the introduction of The Cotton Gin. This machine revolutionized the way cotton was harvested. Another aspect was the stabilization of a larger slave population in the South. Cotton is labor intensive and the man power was available through the use of African slaves. Thirdly markets became larger for the cotton. Trade continued to increase within North America as well as with Europe.

The Praxis® Study Companion 27 Step 4: Determine Your Strategy for Success

4. Determine Your Strategy for Success

Set clear goals and deadlines so your test preparation is focused and efficient

EffectivePraxis test preparation doesn’t just happen. You’ll want to set clear goals and deadlines for yourself along the way. Otherwise, you may not feel ready and confident on test day.

1) Learn what the test covers. You may have heard that there are several different versions of the same test. It’s true. You may take one version of the test and your friend may take a different version a few months later. Each test has different questions covering the same subject area, but both versions of the test measure the same skills and content knowledge.

You’ll find specific information on the test you’re taking onpage 5, which outlines the content categories that the test measures and what percentage of the test covers each topic. Visit www.ets.org/praxis/ testprep for information on other Praxis tests.

2) Assess how well you know the content. Research shows that test takers tend to overestimate their preparedness—this is why some test takers assume they did well and then find out they did not pass.

The Praxis tests are demanding enough to require serious review of likely content, and the longer you’ve been away from the content, the more preparation you will most likely need. If it has been longer than a few months since you’ve studied your content area, make a concerted effort to prepare.

3) Collect study materials. Gathering and organizing your materials for review are critical steps in preparing for the Praxis tests. Consider the following reference sources as you plan your study:

• Did you take a course in which the content area was covered? If yes, do you still have your books or your notes? • Does your local library have a high school-level textbook in this area? Does your college library have a good introductory college-level textbook in this area? Practice materials are available for purchase for many Praxis tests at www.ets.org/praxis/testprep. Test preparation materials include sample questions and answers with explanations.

4) Plan and organize your time. You can begin to plan and organize your time while you are still collecting materials. Allow yourself plenty of review time to avoid cramming new material at the end. Here are a few tips:

• Choose a test date far enough in the future to leave you plenty of preparation time. Test dates can be found at www.ets.org/praxis/register/dates_centers. • Work backward from that date to figure out how much time you will need for review. • Set a realistic schedule—and stick to it.

The Praxis® Study Companion 28 Step 4: Determine Your Strategy for Success

5) Practice explaining the key concepts. Praxis tests with constructed-response questions assess your ability to explain material effectively. As a teacher, you’ll need to be able to explain concepts and processes to students in a clear, understandable way. What are the major concepts you will be required to teach? Can you explain them in your own words accurately, completely, and clearly? Practice explaining these concepts to test your ability to effectively explain what you know.

6) Understand how questions will be scored. Scoring information can be found on page 48.

7) Develop a study plan. A study plan provides a road map to prepare for the Praxis tests. It can help you understand what skills and knowledge are covered on the test and where to focus your attention. Use the study plan template on page 33 to organize your efforts.

And most important—get started!

Would a Study Group Work for You?

Using this guide as part of a study group

People who have a lot of studying to do sometimes find it helpful to form a study group with others who are working toward the same goal. Study groups give members opportunities to ask questions and get detailed answers. In a group, some members usually have a better understanding of certain topics, while others in the group may be better at other topics. As members take turns explaining concepts to one another, everyone builds self-confidence.

If the group encounters a question that none of the members can answer well, the group can go to a teacher or other expert and get answers efficiently. Because study groups schedule regular meetings, members study in a more disciplined fashion. They also gain emotional support. The group should be large enough so that multiple people can contribute different kinds of knowledge, but small enough so that it stays focused. Often, three to six members is a good size.

Here are some ways to use this guide as part of a study group:

• Plan the group’s study program. Parts of the study plan template, beginning on page 33, can help to structure your group’s study program. By filling out the first five columns and sharing the worksheets, everyone will learn more about your group’s mix of abilities and about the resources, such as textbooks, that members can share with the group. In the sixth column (“Dates I will study the content”), you can create an overall schedule for your group’s study program.

• Plan individual group sessions. At the end of each session, the group should decide what specific topics will be covered at the next meeting and who will present each topic. Use the topic headings and subheadings in the Test at a Glance table on page 5 to select topics, and then select practice questions, beginning on page 14.

• Prepare your presentation for the group. When it’s your turn to present, prepare something that is more than a lecture. Write two or three original questions to pose to the group. Practicing writing actual questions can help you better understand the topics covered on the test as well as the types of questions you will encounter on the test. It will also give other members of the group extra practice at answering questions.

The Praxis® Study Companion 29 Step 4: Determine Your Strategy for Success

• Take a practice test together. The idea of a practice test is to simulate an actual administration of the test, so scheduling a test session with the group will add to the realism and may also help boost everyone’s confidence. Remember, complete the practice test using only the time that will be allotted for that test on your administration day.

• Learn from the results of the practice test. Review the results of the practice test, including the number of questions answered correctly in each content category. For tests that contain constructed- response questions, look at the Sample Test Questions section, which also contain sample responses to those questions and shows how they were scored. Then try to follow the same guidelines that the test scorers use.

• Be as critical as you can. You’re not doing your study partner(s) any favors by letting them get away with an answer that does not cover all parts of the question adequately. • Be specific. Write comments that are as detailed as the comments about the sample responses. Indicate where and how your study partner(s) are doing an inadequate job of answering the question. Writing notes in the margins of the answer sheet may also help. • Be supportive. Include comments that point out what your study partner(s) got right. Then plan one or more study sessions based on aspects of the questions on which group members performed poorly. For example, each group member might be responsible for rewriting one paragraph of a response in which someone else did an inadequate job.

Whether you decide to study alone or with a group, remember that the best way to prepare is to have an organized plan. The plan should set goals based on specific topics and skills that you need to learn, and it should commit you to a realistic set of deadlines for meeting those goals. Then you need to discipline yourself to stick with your plan and accomplish your goals on schedule.

The Praxis® Study Companion 30 Step 5: Develop Your Study Plan

5. Develop Your Study Plan Develop a personalized study plan and schedule

Planning your study time is important because it will help ensure that you review all content areas covered on the test. Use the sample study plan below as a guide. It shows a plan for the Core Academic Skills for Educators: Reading test. Following that is a study plan template that you can fill out to create your own plan. Use the “Learn about Your Test” and “Test Specifications" information beginning onpage 5 to help complete it. Use this worksheet to: 1. Define Content Areas: List the most important content areas for your test as defined in chapter 1. 2. Determine Strengths and Weaknesses: Identify your strengths and weaknesses in each content area. 3. Identify Resources: Identify the books, courses, and other resources you plan to use for each content area. 4. Study: Create and commit to a schedule that provides for regular study periods. Praxis Test Name (Test Code): Core Academic Skills for Educators: Reading (5712) Test Date: 9/15/18

How well do What Where can I Dates I will Description I know the resources do I find the Date Content covered study the of content content? have/need for resources I completed content (scale 1–5) the content? need? Key Ideas and Details Draw inferences and Middle school College library, implications from the Close reading 3 English middle school 7/15/18 7/15/18 directly stated content textbook teacher of a reading selection Identify summaries or Middle school College library, paraphrases of the main Determining ideas 3 English middle school 7/17/18 7/17/18 idea or primary purpose textbook teacher of a reading selection Identify summaries Middle and College library, or paraphrases of the high school middle and Determining ideas supporting ideas and 3 7/20/18 7/21/18 English high school specific details in a textbook teachers reading selection Craft, Structure, and Language Skills Determine the author’s Middle and College library, attitude toward material high school middle and Interpreting tone 4 7/25/18 7/26/18 discussed in a reading English high school selection textbook teachers Middle and Identify key transition College library, high school Analysis of words and phrases in a middle and 3 English 7/25/18 7/27/18 structure reading selection and high school textbook, how they are used teachers dictionary Identify how a reading College library, High school selection is organized course notes, Analysis of textbook, in terms of cause/effect, 5 high school 8/1/18 8/1/18 structure college course compare/contrast, teacher, college notes problem/solution, etc. professor Determine the role that College library, High school an idea, reference, or course notes, textbook, Author’s purpose piece of information 5 high school 8/1/18 8/1/18 college course plays in an author’s teacher, college notes discussion or argument professor

(continued on next page)

The Praxis® Study Companion 31 Step 5: Develop Your Study Plan

How well do What Where can I Dates Description I know the resources do I find the I will Date Content covered of content content? have/need for resources I study the completed (scale 1–5) the content? need? content Determine whether College library, High school information presented course notes, Language in textbook, in a reading selection 4 high school 8/1/18 8/1/18 different contexts college course is presented as fact or teacher, college notes opinion professor College library, Identify the meanings of High school course notes, Contextual words as they are used in textbook, 2 high school 8/1/18 8/1/18 meaning the context of a reading college course teacher, college selection notes professor College library, High school Understand figurative course notes, Figurative textbook, language and nuances in 2 high school 8/8/18 8/8/18 language college course word meanings teacher, college notes professor Understand a range College library, High school of words and phrases course notes, textbook, Vocabulary range sufficient for reading at 2 high school 8/15/18 8/17/18 college course the college and career teacher, college notes readiness level professor Integration of Knowledge and Ideas Analyze content College library, presented in diverse High school course notes, Diverse media and media and formats, textbook, 2 high school 8/22/18 8/24/18 formats including visually and college course teacher, college quantitatively, as well as notes professor in words College library, High school Identify the relationship course notes, Evaluation of textbook, among ideas presented 4 high school 8/24/18 8/24/18 arguments college course in a reading selection teacher, college notes professor Determine whether College library, High school evidence strengthens, course notes, Evaluation of textbook, weakens, or is relevant 3 high school 8/27/18 8/27/18 arguments college course to the arguments in a teacher, college notes reading selection professor College library, Determine the logical High school course notes, Evaluation of assumptions upon textbook, 5 high school 8/28/18 8/30/18 arguments which an argument or college course teacher, college conclusion is based notes professor College library, High school Draw conclusions from course notes, Evaluation of textbook, material presented in a 5 high school 8/30/18 8/31/18 arguments college course reading selection teacher, college notes professor Recognize or predict College library, ideas or situations that High school course notes, Comparison of are extensions of or textbook, 4 high school 9/3/18 9/4/18 texts similar to what has been college course teacher, college presented in a reading notes professor selection College library, High school Apply ideas presented course notes, Comparison of textbook, in a reading selection to 2 high school 9/5/18 9/6/18 texts college course other situations teacher, college notes professor

The Praxis® Study Companion 32 Step 5: Develop Your Study Plan

My Study Plan Use this worksheet to: 1. Define Content Areas: List the most important content areas for your test as defined in chapter 1. 2. Determine Strengths and Weaknesses: Identify your strengths and weaknesses in each content area. 3. Identify Resources: Identify the books, courses, and other resources you plan to use for each content area. 4. Study: Create and commit to a schedule that provides for regular study periods.

Praxis Test Name (Test Code): ______Test Date: ______

How well do What Where can I Dates I will Description I know the resources do I find the Date Content covered study this of content content? have/need for resources I completed content (scale 1–5) this content? need?

(continued on next page)

The Praxis® Study Companion 33 Step 5: Develop Your Study Plan

How well do What Where can I Dates I will Description I know the resources do I find the Date Content covered study the of content content? have/need for resources I completed content (scale 1–5) the content? need?

The Praxis® Study Companion 34 Step 6: Review Study Topics

6. Review Study Topics

Review study topics with questions for discussion

Using the Study Topics That Follow The Middle School: Social Studies test is designed to measure the knowledge and skills necessary for a beginning teacher.

This chapter is intended to help you organize your preparation for the test and to give you a clear indication of the depth and breadth of the knowledge required for success on the test.

Virtually all accredited programs address the topics covered by the test; however, you are not expected to be an expert on all aspects of the topics that follow.

You are likely to find that the topics that follow are covered by most introductory textbooks. Consult materials and resources, including lecture and laboratory notes, from all your coursework. You should be able to match up specific topics and subtopics with what you have covered in your courses.

Try not to be overwhelmed by the volume and scope of content knowledge in this guide. Although a specific term may not seem familiar as you see it here, you might find you can understand it when applied to a real-life situation. Many of the items on the actual test will provide you with a context to apply to these topics or terms.

Discussion Areas Interspersed throughout the study topics are discussion areas, presented as open-ended questions or statements. These discussion areas are intended to help test your knowledge of fundamental concepts and your ability to apply those concepts to situations in the classroom or the real world. Most of the areas require you to combine several pieces of knowledge to formulate an integrated understanding and response. If you spend time on these areas, you will gain increased understanding and facility with the subject matter covered on the test. You may want to discuss these areas and your answers with a teacher or mentor.

Note that this study companion does not provide answers for the discussion area questions, but thinking about the answers to them will help improve your understanding of fundamental concepts and will probably help you answer a broad range of questions on the test.

The Praxis® Study Companion 35 Step 6: Review Study Topics

Study Topics 12. Understands social and political changes in the United States from the 1940s to the 1970s An overview of the areas covered on the test, along (e.g., Civil Rights movement, counterculture, with their subareas, follows. conservatism) 13. Understands the impact of economic and technological changes in the latter half of the I. United States History 20th century (e.g., globalization, information A. Chronological Developments in United age) States History B. Major Themes in United States History 1. Understands basic North American geography 1. Understands ongoing impacts of racial, and the peoples and cultures of North gender, and ethnic divisions throughout America prior to European colonization American history 2. Understands how and why European colonies 2. Understands how participants in the political in North America were founded and process (e.g., presidents, the judiciary, developed Congress, and political parties) engaged in 3. Understands how European, African, and politics and shaped policy American Indian peoples interacted in North 3. Understands the emergence of the United America during the colonial period States as a world power and the evolving role 4. Understands the origins of the American of the United States in the world Revolution and the challenges faced by the 4. Understands major economic transformations early republic (e.g., in agriculture, business, and labor) that 5. Understands the context that led to the occurred in the United States writing and adoption of the United States 5. Understands causes and effects of changing Constitution and Bill of Rights and their patterns of immigration to the United States impact on the political development of the and internal migration within the United early United States States 6. Understands causes (e.g., Manifest Destiny, 6. Understands the struggle and achievements gold rush) and effects (e.g., displacement of by individuals and groups for greater political native peoples, economic expansion) of and civil rights throughout United States territorial expansion of the United States in history the 19th century 7. Understands causes and effects of 19th- Discussion Areas: United States History century sectionalism, the Civil War, and • Make your own timeline of United States Reconstruction history, starting with space for each century: 8. Understands the relationship among 1400’s, 1500’s, 1600’s, etc. (recognizing, of industrialization, urbanization, and course, that Native Americans lived on the immigration in the late 19th and early 20th continent for thousands of years before that). centuries Put the events listed in the study topics on your timeline in the correct century, then 9. Knows major political, economic, social, and trace and describe in your own words cultural developments from the Progressive important trends in cultural, intellectual, Era through the New Deal social, economic, political, and diplomatic 10. Understands causes of United States history. participation in the First and Second World • Other trends to identify and describe in your Wars and effects of the wars at home and timeline: abroad –– Migration—patterns and effects 11. Understands the origins, development, and –– Technology—important developments effects of the Cold War both abroad and and their effects domestically –– Urbanization—patterns and effects –– Religions—dominant religions, conflicts with each other and with government, influence on society and politics

The Praxis® Study Companion 36 Step 6: Review Study Topics

–– The emergence of the United States as a • Describe the roles of government leaders, world leader in the areas of military compromises, and events regarding the power, industry, finance, and politics growth of sectionalism in the United States.

• Describe the history of North America before • Describe the main figures, arguments, the settlement of European peoples, practices, and impacts of abolitionism in the including the migration and settlement of antebellum United States. Native American groups and their cultures. • What were the causes, events, and impacts • Describe interactions among European of the Civil War? settlers and Native American groups during the colonial period and developments in • What were the successes and failures of Native American and United States relations Reconstruction and the effects it had on throughout history. society, states, the federal government, and individuals? • What economic factors attracted Europeans to North America? • Describe the push- and pull-factors that contributed to late nineteenth century • Describe the Spanish, French, and English immigration to the United States. colonies in North America • Describe the reform efforts by Susan B. • What principles are included in the Anthony, W. E. B. Dubois, and Robert Declaration of Independence, the Articles of LaFollete and their impacts on United States Confederation, the United States society. Constitution, the Bill of Rights, and how did these documents affect the development of • Compare and contrast populism and the United States? progressivism

• Describe major debates, compromises, and • What were the major successes, failures, and questions raised by the United States legacies of the New Deal? Constitution such as The Great Compromise, the Three-Fifths Compromise, and the • Describe the United States involvement in federalist and anti-federalist arguments. foreign wars during the 20th century, including the Cold War era and events of the • What were the positions of the Founding late 20th and early 21st centuries (e.g., The Fathers regarding slavery? First and Second World Wars, The Korean War, The Vietnam War, the wars in Iraq and • What were the political and economic Afghanistan). causes and outcomes of the War of 1812? • What were the roles of technology, • What was the influence of Jacksonian education, domestic policy, and the Democracy on the society, political climate, economy on United States society and economy of the United States? throughout the 20th and early 21st centuries? • Describe manifest destiny and the expansion of the United States.

• What was the impact of westward expansion II. World History on the United States economy? A. Chronological Developments in World History • What were the long-term trends or developments that contributed to the 1. Knows the formation, organization, and growth of sectionalism? contributions of early civilizations in Africa, Europe, and Asia from 8000 B.C.E. to 1000 • Describe the successes and failures of B.C.E. and in the Americas from 2000 B.C.E. to attempts to solve regional political 1500 C.E. differences.

The Praxis® Study Companion 37 Step 6: Review Study Topics

2. Knows the formation, organization, and 7. Knows major demographic trends (e.g., contributions of the classical civilizations of urbanization, migration, population growth) in Greece, Rome, Persia, India, and China from world history and their effects 1000 B.C.E. to 500 C.E. 8. Knows how exploration (e.g., continental, 3. Knows how civilizations changed as a result of overseas, outer space) affects human societies invasions, trade, and the spread of Islam and 9. Understands how struggles for human rights Christianity from 300 to 1400 C.E. and individual expression shaped various 4. Knows causes and effects of the Renaissance, societies Reformation, and Enlightenment from 1400 to 1800 C.E. Discussion areas: World History 5. Knows major causes and effects of growing • Work with a globe or world map as you study global interactions from 1200 to 1750 C.E. and review world history. It would be (e.g., Columbian Exchange, colonization, the especially useful to use a historical atlas so Plague) that you can see a place or region in its historical context. In addition, recent world 6. Knows major causes and effects of history textbooks have many excellent maps. revolutions, nationalism, and imperialism from Find regions and places you are studying on 1750 to 1914 the globe and make sure you understand the 7. Knows major ideological, economic, and locations, movements, and relationships political causes and effects of the First and among the many societies you are reviewing. Second World Wars and the Cold War (e.g., Russian Revolution, decolonization) • Think carefully about the periods into which this history is divided. You will probably find 8. Knows major developments of the post–Cold alternative schemes—that is, different names War world (e.g., growth of the globalized and year spans—in the materials you use for economy, rise of fundamentalism and review. Why do historians divide history into nationalism) periods? Do they agree on the names and B. Major Themes in World History dates of some periods more than others? What do the periods say about historical 1. Understands how scientific inquiry, interpretation? How do periods relate to technological innovations, and adaptations long-term trends? shaped world societies 2. Understands the role of major world religions • Explain the significance of the following in shaping societies and effecting major dates in world history: 220 C.E. and 476 C.E., historical turning points (e.g., the spread of 622 C.E., 1096 1099 C.E., 1200–1300, 1453, Buddhism, the Crusades, Hinduism) 1492, 1750–1780, 1789, 1870’s, 1914–1918, 1939–1945, 1947, 1957, 1989. 3. Knows the role of trade and other forms of economic exchange (e.g., tribute, gift giving) • Describe the concepts of citizenship and both within societies and in contacts between democracy in ancient Greece and their societies similarities and differences in contemporary 4. Knows major political ideologies that have democracies. influenced the organization of societies in the modern world (e.g., fascism, liberalism, • Describe the rise and fall of the Roman nationalism) Empire, its successes and failures and its contributions to the development of 5. Understands major economic transformations Western society. that have affected world societies (e.g., feudalism, spread of the market economy, • Describe the developments in the social, industrialization) cultural, economic, and political histories of 6. Understands the roles of both conflict (e.g., societies in the Indian subcontinent (e.g., imperialism) and cooperation (e.g., United Hinduism and Buddhism, the caste system, Nations) in shaping and transforming societies interactions with other societies).

The Praxis® Study Companion 38 Step 6: Review Study Topics

• What fundamental ideas and institutions 3. Understands the origins, development, arose from the cultures of India, China, Egypt, interpretations, and ongoing impact of the Greece, Rome, and Mesopotamia? United States Constitution 4. Understands the origin, role, and interactions • Describe the role of Islam in Middle East and of the three branches of the federal African history and its influence in Europe government and Asia. 5. Understands the development and political • Describe the interactions between the impact of civil rights and civil liberties in the Islamic world and Medieval Europe during United States the Crusades. 6. Understands how the election process • Describe Mesoamerican cultures and their operates in the United States interactions with early European explorers. 7. Understands the role of political parties, interest groups and voluntary associations, • Describe European societal changes from and the media in the political process the medieval period to the Renaissance and the Age of Enlightenment (e.g., the 8. Understands the rights, responsibilities, and Reformation, the Scientific Revolution). duties of citizens B. Comparative Government and • How did Enlightenment ideas affect the International Relations American, French, and Haitian Revolutions? 1. Understands the major characteristics of • Describe the onset of the first Industrial different political systems (e.g., democracy, Revolution, the conditions that led to its republic, totalitarianism) emergence, and its effects on global society. 2. Understands the relationship between • What were the causes, major events, and political systems and economic conditions effects of major political revolutions and Discussion areas: Government/Civics independence movements (The American Revolution, the French Revolution, • Describe the main ideas of political theorists revolutions in Latin America)? and their contributions to the development of forms of government and their institutions • Describe the emergence of totalitarian (e.g., Locke, Hobbes, Montesquieu). governments after the First World War. • Describe the constitutional interpretations in • What were the effects of Western nationalism landmark Supreme Court cases (e.g., Marbury and imperialism? v. Madison, McCulloch v. , Brown v. Board of Education of Topeka, Miranda v. • What were the similarities and differences Arizona). among the Russian, Mexican, and Chinese revolutions? • What are the effects of bicameralism on the • Describe the emergence of a global culture exercise of legislative power? in the late twentieth century and its major • What are the major differences between the elements and consequences. two chambers of Congress?

III. Government/Civics • Describe presidential powers. A. United States Government and Civics • Describe the regulatory functions of 1. Understands the key concepts and ideas on government agencies and independent which the United States government is based regulatory commissions. (e.g., popular sovereignty, separation of • How do institutions affect the political powers, rule of law) process of the United States? 2. Understands federalism and the basic relationship between the states and the • Describe the influence of race, gender, class, national government and age on public opinion, individual beliefs, and political participation.

The Praxis® Study Companion 39 Step 6: Review Study Topics

• What are the major differences between Discussion areas: Geography parliamentary and presidential systems? • What are the primary characteristics of each of the following regions: North Africa/ • How does a country’s electoral system affects Southwest Asia, Sub-Saharan Africa, Latin its political parties? America, the Caribbean, North America, • Describe some basic approaches to foreign Western Europe, Eastern Europe, East Asia, policy (e.g., isolationism, multilateralism). South Asia, Southeast Asia, and Oceania?

• Describe the major problems that • What is the difference between weather and international organizations face. climate? • How does each of the following factors IV. Geography influence climate: Latitude, ocean currents, winds, mountains, elevation, proximity to A. Geographic Literacy water? 1. Understands relative and absolute location and the physical and human characteristics of • What are the patterns of natural occurrences “place” and why do they occur in certain areas? 2. Understands how to utilize characteristics to • What is an ecosystem and why is identify regions understanding ecosystems important? B. Map Skills • Be able to read and interpret population 1. Understands and interprets various types of pyramids maps • What regions of the United States grew more 2. Understands latitude and longitude rapidly than others in the 20th and 21st 3. Understands spacial patterns and their centuries, and why? meanings • What are the major trends in ethnic 4. Is familiar with recent technological sources of composition of the United States population geographic information in the twentieth and twenty-first centuries? C. Physical Geography • How does having a wide range of climate 1. Demonstrates knowledge of various types of zones in Latin America influence the history physical features on the Earth’s surface of the region? 2. Understands the interrelationship between the environment and human activity • How has having a limited supply of water influenced the historical and economic 3. Knows the physical processes related to the development of the Middle East? formation of geographic features 4. Knows the basic patterns of climate, • How do major human alterations of the ecosystems, and natural resources landscape, such as the Panama and Suez Canals, affect economic, political, and D. Human Geography cultural history? 1. Knows spatial patterns of culture and economic activities • Name some changes that have occurred in the twentieth century in the use, distribution, 2. Understands basic population patterns and importance of natural resources. 3. Understands basic patterns of land use 4. Understands patterns and interrelationships of migration, trade, and diffusion of ideas 5. Knows basic concepts of political geography 6. Understands how culture and cultural change, human adaptation, and diversity influence human behavior

The Praxis® Study Companion 40 Step 6: Review Study Topics

V. Economics • What are the sources of gain from trade? A. Microeconomics I • What are the methods of economic 1. Knows the concepts of scarcity, choice, and organization and how do they differ? opportunity costs • Explain the four sectors contained in a model 2. Knows the differences among market, of circular flow of income and products and command, and mixed economies how the circular-flow model describes the B. Microeconomics II operation of the market economy 1. Demonstrates knowledge of types of market • What are the laws of supply and demand? structure and the characteristics and behavior of firms in perfect competition, monopoly, • How is the market price of a good oligopoly, and monopolistic competition determined

2. Demonstrates understanding of factor • How do markets adjust to changes in supply markets and the determination of income and demand distributions and the returns to factors of production (wage, interest, rent, profits) • What is the relationship between total C. Macroeconomics I revenue and the price elasticity of demand? 1. Demonstrates understanding of gross • How does the imposition of a tax affect a domestic product (GDP) and its components market? 2. Demonstrates understanding of how • Define total product, average product, and unemployment is measured and its causes marginal product and consequences 3. Understands the principle of supply and • What is the law of diminishing returns? demand • What are explicit costs and implicit costs? 4. Knows how government intervention and public policies affect the economy (e.g., • Define the following costs: total cost, fixed taxation, regulation) costs, variable costs, average total cost, 5. Knows how consumer choices affect the average fixed cost, average variable cost, and economy marginal cost. 6. Understands basic principles of personal • What are the characteristics of perfect finance (e.g., budgeting, interest, investment) competition, monopoly, monopolistic 7. Understands how the factors of production competition, and oligopoly? (i.e., land, labor, capital, and entrepreneurship) • How do government policies attempt to affect economic activity regulate monopolies 8. Knows different measures of economic performance (e.g., GDP, unemployment, • Why do business firms demand labor, inflation) machines, and other resources? 9. Understands basic functions of money and • What are the determinants of labor demand? how it circulates through the economy • What determines the market price of a 10. Knows the interdependence of the global resource such as labor? economy • Why do some people earn more than others Discussion areas: Economics • Why does scarcity force people to consider • What is the effect of the minimum-wage law opportunity cost? on wages and employment in the labor markets? • What do production possibilities curves demonstrate? • What are the major components of gross domestic product (G D P) and how is G D P • Why do people engage in exchange? measured in the United States?

The Praxis® Study Companion 41 Step 6: Review Study Topics

• What do price indices measure? • What is the relationship between a country’s net exports and the international value of • What are the causes and costs of inflation? the country’s currency?

• How is unemployment measured and how • What is the role of productivity in economic does the unemployment rate understate or growth? overstate the amount of joblessness? • What policies can a government pursue to • Name the different types of unemployment raise the living standards of a country in the long run? • How do you determine the equilibrium level of G D P of any economy? VI. Short Content Essays • What are aggregate demand curves and the factors that cause shifts in aggregate The three equally weighted short-answer/essay demand? questions are designed to test knowledge of history and social studies and the ability to define and clarify • How do economies adjust to changes in issues, judge information, and draw conclusions. aggregate supply and aggregate demand? In most cases, questions ask examinees to utilize • What are the causes of recessions and these abilities in demonstrating an understanding booms? of stimulus materials such as maps, graphs, charts, • What are the three functions of money, the tables, cartoons, diagrams, quotations, or excerpts measures of money supply, and the different from documents. kinds of money demand? Each of the three questions in the test will have both • What are the major functions of the Federal a history and a social studies component. Reserve System? The three questions will be divided as follows: • What is the role of banks in the money creation process? 1. United States History, related to Government/Civics

• What are the important tools of the Federal 2. World History, related to Geography Reserve for controlling the supply of money? 3. United States History, related to Economics or Government/Civics • What is appropriate fiscal or monetary policies for dealing with the following? 4. World History, related to Economics or Government/Civics –– Inflation –– Recessions OR –– Stagflation 1. United States History, related to Government/Civics • Why do nations trade and what can nations 2. World History, related to Geography can gain from international trade? 3. United States History, related to Economics OR • Why do nations impose trade restrictions Geography and what impacts do trade restrictions have on the economy? OR

• What are the pros and cons of protectionist 1. United States History, related to Government/Civics policies? 2. World History, related to Geography

• What determines the exchange rate of the 3. World History, related to Economics OR United States dollar in the foreign-exchange Government/Civics market?

• What information is provided in the balance of payments account?

The Praxis® Study Companion 42 Step 7: Review Smart Tips for Success

7. Review Smart Tips for Success

Follow test-taking tips developed by experts

Learn from the experts. Take advantage of the following answers to questions you may have and practical tips to help you navigate the Praxis test and make the best use of your time.

Should I guess? Yes. Your score is based on the number of questions you answer correctly, with no penalty or subtraction for an incorrect answer. When you don’t know the answer to a question, try to eliminate any obviously wrong answers and then guess at the correct one. Try to pace yourself so that you have enough time to carefully consider every question.

Can I answer the questions in any order? You can answer the questions in order or skip questions and come back to them later. If you skip a question, you can also mark it so that you can remember to return and answer it later. Remember that questions left unanswered are treated the same as questions answered incorrectly, so it is to your advantage to answer every question.

Are there trick questions on the test? No. There are no hidden meanings or trick questions. All of the questions on the test ask about subject matter knowledge in a straightforward manner.

Are there answer patterns on the test? No. You might have heard this myth: the answers on tests follow patterns. Another myth is that there will never be more than two questions in a row with the correct answer in the same position among the choices. Neither myth is true. Select the answer you think is correct based on your knowledge of the subject.

Can I write on the scratch paper I am given? Yes. You can work out problems on the scratch paper, make notes to yourself, or write anything at all. Your scratch paper will be destroyed after you are finished with it, so use it in any way that is helpful to you. But make sure to select or enter your answers on the computer.

Smart Tips for Taking the Test 1. Skip the questions you find extremely difficult. Rather than trying to answer these on your first pass through the test, you may want to leave them blank and mark them so that you can return to them later. Pay attention to the time as you answer the rest of the questions on the test, and try to finish with 10 or 15 minutes remaining so that you can go back over the questions you left blank. Even if you don’t know the answer the second time you read the questions, see if you can narrow down the possible answers, and then guess. Your score is based on the number of right answers, so it is to your advantage to answer every question.

The Praxis® Study Companion 43 Step 7: Review Smart Tips for Success

2. Keep track of the time. The on-screen clock will tell you how much time you have left. You will probably have plenty of time to answer all of the questions, but if you find yourself becoming bogged down, you might decide to move on and come back to any unanswered questions later.

3. Read all of the possible answers before selecting one. For questions that require you to select more than one answer, or to make another kind of selection, consider the most likely answers given what the question is asking. Then reread the question to be sure the answer(s) you have given really answer the question. Remember, a question that contains a phrase such as “Which of the following does NOT …” is asking for the one answer that is NOT a correct statement or conclusion.

4. Check your answers. If you have extra time left over at the end of the test, look over each question and make sure that you have answered it as you intended. Many test takers make careless mistakes that they could have corrected if they had checked their answers.

5. Don’t worry about your score when you are taking the test. No one is expected to answer all of the questions correctly. Your score on this test is not analogous to your score on the GRE® or other tests. It doesn’t matter on the Praxis tests whether you score very high or barely pass. If you meet the minimum passing scores for your state and you meet the state’s other requirements for obtaining a teaching license, you will receive a license. In other words, what matters is meeting the minimum passing score. You can find passing scores for all states that use the Praxis tests at https://www.ets.org/praxis/institutions/scores/passing/ or on the web site of the state for which you are seeking certification/licensure.

6. Use your energy to take the test, not to get frustrated by it. Getting frustrated only increases stress and decreases the likelihood that you will do your best. Highly qualified educators and test development professionals, all with backgrounds in teaching, worked diligently to make the test a fair and valid measure of your knowledge and skills. Your state painstakingly reviewed the test before adopting it as a licensure requirement. The best thing to do is concentrate on answering the questions.

The Praxis® Study Companion 44 Step 8: Check on Testing Accommodations

8. Check on Testing Accommodations See if you qualify for accommodations to take the Praxis test

What if English is not my primary language? Praxis tests are given only in English. If your primary language is not English (PLNE), you may be eligible for extended testing time. For more details, visit www.ets.org/praxis/register/plne_accommodations/.

What if I have a disability or other health-related need? The following accommodations are available for Praxis test takers who meet the Americans with Disabilities Act (ADA) Amendments Act disability requirements:

• Extended testing time • Additional rest breaks • Separate testing room • Writer/recorder of answers • Test reader • Sign language interpreter for spoken directions only • Perkins Brailler • Braille slate and stylus • Printed copy of spoken directions • Oral interpreter • Audio test • Braille test • Large print test book • Large print answer sheet • Listening section omitted

For more information on these accommodations, visit www.ets.org/praxis/register/disabilities.

Note: Test takers who have health-related needs requiring them to bring equipment, beverages, or snacks into the testing room or to take extra or extended breaks must request these accommodations by following the procedures described in the Bulletin Supplement for Test Takers with Disabilities or Health-Related Needs (PDF), which can be found at https://www.ets.org/s/praxis/pdf/bulletin_supplement_test_takers_with_ disabilities_health_needs.pdf.

You can find additional information on available resources for test takers with disabilities or health-related needs at www.ets.org/disabilities.

The Praxis® Study Companion 45 Step 9: Do Your Best on Test Day

9. Do Your Best on Test Day

Get ready for test day so you will be calm and confident

You followed your study plan. You prepared for the test. Now it’s time to prepare for test day.

Plan to end your review a day or two before the actual test date so you avoid cramming. Take a dry run to the test center so you’re sure of the route, traffic conditions, and parking. Most of all, you want to eliminate any unexpected factors that could distract you from your ultimate goal—passing the Praxis test!

On the day of the test, you should:

• be well rested • wear comfortable clothes and dress in layers • eat before you take the test • bring an acceptable and valid photo identification with you • bring an approved calculator only if one is specifically permitted for the test you are taking (see Calculator Use, at http://www.ets.org/praxis/test_day/policies/calculators) • be prepared to stand in line to check in or to wait while other test takers check in You can’t control the testing situation, but you can control yourself. Stay calm. The supervisors are well trained and make every effort to provide uniform testing conditions, but don’t let it bother you if the test doesn’t start exactly on time. You will have the allotted amount of time once it does start.

You can think of preparing for this test as training for an athletic event. Once you’ve trained, prepared, and rested, give it everything you’ve got.

What items am I restricted from bringing into the test center? You cannot bring into the test center personal items such as:

• handbags, knapsacks, or briefcases • water bottles or canned or bottled beverages • study materials, books, or notes • pens, pencils, scrap paper, or calculators, unless specifically permitted for the test you are taking (see Calculator Use, at http://www.ets.org/praxis/test_day/policies/calculators) • any electronic, photographic, recording, or listening devices

Personal items are not allowed in the testing room and will not be available to you during the test or during breaks. You may also be asked to empty your pockets. At some centers, you will be assigned a space to store your belongings, such as handbags and study materials. Some centers do not have secure storage space available, so please plan accordingly.

Test centers assume no responsibility for your personal items.

The Praxis® Study Companion 46 Step 9: Do Your Best on Test Day

If you have health-related needs requiring you to bring equipment, beverages or snacks into the testing room or to take extra or extended breaks, you need to request accommodations in advance. Procedures for requesting accommodations are described in the Bulletin Supplement for Test Takers with Disabilities or Health-related Needs (PDF).

Note: All cell phones, smart phones (e.g., Android® devices, iPhones®, etc.), and other electronic, photographic, recording, or listening devices are strictly prohibited from the test center. If you are seen with such a device, you will be dismissed from the test, your test scores will be canceled, and you will forfeit your test fees. If you are seen using such a device, the device will be confiscated and inspected. For more information on what you can bring to the test center, visit www.ets.org/praxis/test_day/bring.

Are You Ready? Complete this checklist to determine whether you are ready to take your test. ❒ Do you know the testing requirements for the license or certification you are seeking in the state(s) where you plan to teach? ❒ Have you followed all of the test registration procedures? ❒ Do you know the topics that will be covered in each test you plan to take? ❒ Have you reviewed any textbooks, class notes, and course readings that relate to the topics covered? ❒ Do you know how long the test will take and the number of questions it contains? ❒ Have you considered how you will pace your work? ❒ Are you familiar with the types of questions for your test? ❒ Are you familiar with the recommended test-taking strategies? ❒ Have you practiced by working through the practice questions in this study companion or in a study guide or practice test? ❒ If constructed-response questions are part of your test, do you understand the scoring criteria for these questions? ❒ If you are repeating a Praxis test, have you analyzed your previous score report to determine areas where additional study and test preparation could be useful?

If you answered “yes” to the questions above, your preparation has paid off. Now take thePraxis test, do your best, pass it—and begin your teaching career!

The Praxis® Study Companion 47 Step 10: Understand Your Scores

10. Understand Your Scores Understand how tests are scored and how to interpret your test scores

Of course, passing the Praxis test is important to you so you need to understand what your scores mean and what your state requirements are.

What are the score requirements for my state? States, institutions, and associations that require the tests set their own passing scores. Visit www.ets.org/praxis/states for the most up-to-date information.

If I move to another state, will my new state accept my scores? The Praxis tests are part of a national testing program, meaning that they are required in many states for licensure. The advantage of a national program is that if you move to another state that also requires Praxis tests, you can transfer your scores. Each state has specific test requirements and passing scores, which you can find at www.ets.org/praxis/states.

How do I know whether I passed the test? Your score report will include information on passing scores for the states you identified as recipients of your test results. If you test in a state with automatic score reporting, you will also receive passing score information for that state.

A list of states and their passing scores for each test are available online at www.ets.org/praxis/states.

What your Praxis scores mean You received your score report. Now what does it mean? It’s important to interpret your score report correctly and to know what to do if you have questions about your scores.

Visit http://www.ets.org/s/praxis/pdf/sample_score_report.pdf to see a sample score report. To access Understanding Your Praxis Scores, a document that provides additional information on how to read your score report, visit www.ets.org/praxis/scores/understand.

Put your scores in perspective Your score report indicates: • Your score and whether you passed • The range of possible scores • The raw points available in each content category • The range of the middle 50 percent of scores on the test If you have taken the same Praxis test or other Praxis tests in the last 10 years, your score report also lists the highest score you earned on each test taken.

The Praxis® Study Companion 48 Step 10: Understand Your Scores

Content category scores and score interpretation Questions on the Praxis tests are categorized by content. To help you in future study or in preparing to retake the test, your score report shows how many raw points you earned in each content category. Compare your “raw points earned” with the maximum points you could have earned (“raw points available”). The greater the difference, the greater the opportunity to improve your score by further study.

Score scale changes ETS updates Praxis tests on a regular basis to ensure they accurately measure the knowledge and skills that are required for licensure. When tests are updated, the meaning of the score scale may change, so requirements may vary between the new and previous versions. All scores for previous, discontinued tests are valid and reportable for 10 years, provided that your state or licensing agency still accepts them.

These resources may also help you interpret your scores:

• Understanding Your Praxis Scores (PDF), found at www.ets.org/praxis/scores/understand

• The Praxis Passing Scores, found at https://www.ets.org/praxis/institutions/scores/passing/

• State requirements, found at www.ets.org/praxis/states

The Praxis® Study Companion 49 Appendix: Other Questions You May Have

Appendix: Other Questions You May Have

Here is some supplemental information that can give you a better understanding of the Praxis tests.

What do the Praxis tests measure? The Praxis tests measure the specific knowledge and skills that beginning teachers need. The tests do not measure an individual’s disposition toward teaching or potential for success, nor do they measure your actual teaching ability. The assessments are designed to be comprehensive and inclusive but are limited to what can be covered in a finite number of questions and question types. Teaching requires many complex skills that are typically measured in other ways, including classroom observation, video recordings, and portfolios.

Ranging from Agriculture to World Languages, there are more than 80 Praxis tests, which contain selected- response questions or constructed-response questions, or a combination of both.

Who takes the tests and why? Some colleges and universities use the Praxis Core Academic Skills for Educators tests (Reading, Writing, and Mathematics) to evaluate individuals for entry into programs. The assessments are generally taken early in your college career. Many states also require Core Academic Skills test scores as part of their teacher licensing process.

Individuals entering the teaching profession take the Praxis content and pedagogy tests as part of the teacher licensing and certification process required by many states. In addition, some professional associations and organizations require Praxis Subject Assessments for professional licensing.

Do all states require these tests? The Praxis tests are currently required for teacher licensure in approximately 40 states and United States territories. These tests are also used by several professional licensing agencies and by several hundred colleges and universities. Teacher candidates can test in one state and submit their scores in any other state that requires Praxis testing for licensure. You can find details atwww.ets.org/praxis/states .

What is licensure/certification? Licensure in any area—medicine, law, architecture, accounting, cosmetology—is an assurance to the public that the person holding the license possesses sufficient knowledge and skills to perform important occupational activities safely and effectively. In the case of teacher licensing, a license tells the public that the individual has met predefined competency standards for beginning teaching practice.

Because a license makes such a serious claim about its holder, licensure tests are usually quite demanding. In some fields, licensure tests have more than one part and last for more than one day. Candidates for licensure in all fields plan intensive study as part of their professional preparation. Some join study groups, others study alone. But preparing to take a licensure test is, in all cases, a professional activity. Because a licensure exam surveys a broad body of knowledge, preparing for a licensure exam takes planning, discipline, and sustained effort.

Why does my state require the Praxis tests? Your state chose the Praxis tests because they assess the breadth and depth of content—called the “domain”— that your state wants its teachers to possess before they begin to teach. The level of content knowledge, reflected in the passing score, is based on recommendations of panels of teachers and teacher educators in

The Praxis® Study Companion 50 Appendix: Other Questions You May Have

each subject area. The state licensing agency and, in some states, the state legislature ratify the passing scores that have been recommended by panels of teachers.

How were the tests developed? E T S consulted with practicing teachers and teacher educators around the country during every step of the Praxis test development process. First, E T S asked them what knowledge and skills a beginning teacher needs to be effective. Their responses were then ranked in order of importance and reviewed by hundreds of teachers.

After the results were analyzed and consensus was reached, guidelines, or specifications, for the selected- response and constructed-response tests were developed by teachers and teacher educators. Following these guidelines, teachers and professional test developers created test questions that met content requirements and E T S Standards for Quality and Fairness.*

When your state adopted the research-based Praxis tests, local panels of teachers and teacher educators evaluated each question for its relevance to beginning teachers in your state. During this “validity study,” the panel also provided a passing-score recommendation based on how many of the test questions a beginning teacher in your state would be able to answer correctly. Your state’s licensing agency determined the final passing-score requirement.

E T S follows well-established industry procedures and standards designed to ensure that the tests measure what they are intended to measure. When you pass the Praxis tests your state requires, you are proving that you have the knowledge and skills you need to begin your teaching career.

How are the tests updated to ensure the content remains current? Praxis tests are reviewed regularly. During the first phase of review, E T S conducts an analysis of relevant state and association standards and of the current test content. State licensure titles and the results of relevant job analyses are also considered. Revised test questions are then produced following the standard test development methodology. National advisory committees may also be convened to review and revise existing test specifications and to evaluate test forms for alignment with the specifications.

How long will it take to receive my scores? Scores for tests that do not include constructed-response questions are available on screen immediately after the test. Scores for tests that contain constructed-response questions or essays aren’t available immediately after the test because of the scoring process involved. Official score reports are available to you and your designated score recipients approximately two to three weeks after the test date for tests delivered continuously, or two to three weeks after the testing window closes for other tests. See the test dates and deadlines calendar at www. ets.org/praxis/register/dates_centers for exact score reporting dates.

Can I access my scores on the web? All test takers can access their test scores via My Praxis Account free of charge for one year from the posting date. This online access replaces the mailing of a paper score report.

The process is easy—simply log into My Praxis Account at www.ets.org/praxis and click on your score report. If you do not already have a Praxis account, you must create one to view your scores.

Note: You must create a Praxis account to access your scores, even if you registered by mail or phone.

*E T S Standards for Quality and Fairness (2014, Princeton, N.J.) are consistent with the Standards for Educational and Psychological Testing, industry standards issued jointly by the American Association, the American Psychological Association, and the National Council on Measurement in Education (2014, Washington, D.C.).

The Praxis® Study Companion 51

Your teaching career is worth preparing for, so start today! Let the Praxis® Study Companion guide you.

To search for the Praxis test prep resources that meet your specific needs, visit: www.ets.org/praxis/testprep

To purchase official test prep made by the creators of the Praxis tests, visit the ETS Store: www.ets.org/praxis/store

Copyright © 2018 by Educational Testing Service. All rights reserved. E T S, the E T S logo, GRE, PRAXIS, and MEASURING THE POWER OF LEARNING are registered trademarks of Educational Testing Service (E T S). All other trademarks are property of their respective owners.